Current

You might also like

Download as pdf or txt
Download as pdf or txt
You are on page 1of 41

17 Current Electricity

Question: The equivalent capacitance of the arrangement shown in figure is:

NEET 2023 Manipur

Answer: D
Explanation

Question: A certain wire A has resistance 81 Ω. The resistance of another


wire B of same material and equal length but of diameter thrice the diameter
of A will be :

NEET 2023 Manipur

Answer: B

Explanation

Question:
Answer: B

Explanation

Question:

NEET 2023 Manipur


Answer: B

Explanation

Question:

Answer: A

Explanation
Question: The magnitude and direction of the current in the following circuit
is :-

NEET 2023

Answer: A

Explanation
Question:

NEET 2023

Answer: B

Explanation
For no reading galvanometer. Potential across it is same.
Question: The equivalent capacitance of the system shown in the following
circuit is:

NEET 2023

Answer: D

Explanation

Question: Resistance of a carbon resistor determined from colour codes


is (22000±5%)Ω. The colour of third band must be :

NEET 2023
Answer: B

Explanation

Question: The resistance of platinum wire at 0∘C is 2 Ω and 6.8 Ω at 80∘C. The
temperature coefficient of resistance of the wire is :

NEET 2023

Answer: B
Explanation

Question:

NEET 2023

Answer: A

Explanation
2022
MCQ (Single Correct Answer)

Q.1. The reciprocal of resistance is :

NEET 2022 Phase 2

Ans. (A)
Explanation

Q.2. The distance between the two plates of a parallel plate capacitor is
doubled and the area of each plate is halved. If C is its initial capacitance,
its final capacitance is equal to
NEET 2022 Phase 2

Ans. (A)
Explanation

Q.3. A cell of emf 4 V and internal resistance 0.5 Ω is connected to a


7.5 Ω external resistance. The terminal potential difference of the cell is

NEET 2022 Phase 2

Ans. (B)
Explanation
Q.4. The equivalent resistance of the infinite network given below is :
NEET 2022 Phase 2

Ans. (D)
Explanation
Let net resistance of the given infinite network be 'R'

Now, the circuit can be modified as

Q.5. The effective capacitances of two capacitors are 3 μF and 16 μF,


when they are connected in series and parallel respectively. The
capacitance of two capacitors are :
NEET 2022 Phase 2

Ans. (D)
Explanation

Q.6. The sliding contact C is at one fourth of the length of the


potentiometer wire (AB) from A as shown in the circuit diagram. If the
resistance of the wire AB is R0, then the potential drop (V) across the
resistor R is
NEET 2022 Phase 2

Ans. (B)
Explanation
Q.7. Two resistors of resistance, 100 Ω and 200 Ω are connected in parallel
in an electrical circuit. The ratio of the thermal energy developed in 100 Ω to
that in 200 Ω in a given time is

NEET 2022 Phase 1

Ans. (B)
Explanation

Q.8. A copper wire of length 10 m and radius m has electrical


resistance of 10 Ω. The current density in the wire for an electric field
strength of 10 (V/m) is
NEET 2022 Phase 1

Ans. (D)
Explanation

Q.9. A capacitor of capacitance C = 900 pF is charged fully by 100 V


battery B as shown in figure (a). Then it is disconnected from the battery
and connected to another uncharged capacitor of capacitance C = 900 pF
as shown in figure (b). The electrostatic energy stored by the system (b) is
NEET 2022 Phase 1

Ans. (C)
Explanation
Q.10. A wheatstone bridge is used to determine the value of unknown
resistance X by adjusting the variable resistance Y as shown in the figure.
For the most precise measurement of X, the resistance P and Q
NEET 2022 Phase 1

Ans. (B)
Explanation
We know, a wheatstone bridge is said to be most precise when it is most
sensitive. This can be done by making ratio arms equal. Thus (2) is correct
option.
TOPIC 1 m
Electrical resistivity, ρ = For series combination,
ne 2 τ R eq = R 1 + R 2 + R 3 + R 4
Ohm’s Law and Rearranging the above expression, ⇒ R eq = R + R + R + R
Resistance τ= 2
Relaxation period,
m
⇒ R eq = 4 R ⇒ R eq = 4(1) = 4 Ω
ne ρ
01 Column I Gives certain physical As we know that, the expression of 03 Three resistors having resistances
terms associated with flow of current density, r1 , r2 and r3 are connected as
current through a metallic I neAv d shown in the given circuit. The
J= = = nev d
conductor. A A i
Column II Gives some E E 1 ratio 3 of currents in terms of
Again, = = =ρ i1
mathematical relations involving J σE σ
electrical quantities. Match E resistances used in the circuit is
∴Electrical resistivity, ρ =
J [NEET 2021]
Column I and Column II with i2 r2
appropriate relations. [NEET 2021] The correct match is A→ 3, B→ 4, C→ 1,
D→ 2. i1 r1
Column I Column II A B
A. Drift velocity 1. m 02 The effective resistance of a
ne 2 ρ parallel connection that consists i3 r3
of four wires of equal length, equal
B. Electrical resistivity 2. nev d r2
area of cross-section and same r1
C. Relaxation period 3. eE (a) (b)
τ material is 0.25 Ω. What will be r2 + r3 r2 + r3
m
the effective resistance if they are r r2
D. Current density 4. E (c) 1 (d)
connected in series? [NEET 2021] r1 + r2 r1 + r3
J
(a) 0.25 Ω (b) 0.5 Ω
Ans. (b)
(c) 1 Ω (d) 4 Ω
Codes According to the given circuit diagram
A B C D Ans. (d) in question,
(a) 3 4 1 2 The four resistances have equal length, i1 = i2 + i3 …(i)
(b) 3 4 2 1 equal cross-sectional area and same
In parallel arrangement of the electrical
(c) 3 1 4 2 material.
circuit,
(d) 3 2 4 1 So, R 1 = R 2 = R 3 = R 4 = R (say)
the voltage remains same.
Ans. (a) Given, the effective resistance,
∴ V = ir (using Ohm’s law)
As we know that, the expression of the R eq = 0.25 Ω
⇒ i 2 r2 = i 3 r3
drift velocity, For parallel arrangement, i r
eE 1 1 1 1 1 ⇒ i2 = 3 3
vd = τ = + + + r2
m R eq R 1 R 2 R3 R4
Here, e is the electric charge, 1 1 1 1 1 Substituting the values in the Eq. (i), we
⇒ = + + + get
E is the electric field, R eq R R R R i r r
i
m is the mass of an electron, 1 4 i1 = 3 3 + i3 ⇒ 1 = 3 + 1
⇒ = r2 i 3 r2
τ is the relaxation time. 0.25 Ω R
i 1 r3 + r2 i r
Consider the conductor having length l, ⇒ = ⇒ 3 = 2
⇒ R = 1Ω i3 r2 i 1 r3 + r2
area of the cross-section A and the
charge density n.
04 Two solid conductors are made up Ans. (a) Ans. (b)
of same material, have same length Given, drift velocity,v d = 7. 5 × 10 −4 m/s Resistivity of copper (a metal) as a
and same resistance. One of them Electric field, E = 3 × 10 −10 V-m function of temperature increases with
has a circular cross-section of Mobility, µ = ? the increase in temperature as shown
v d 7. 5 × 10 −4 below,
area A 1 and the other one has a As, µ= = ρ
square cross-section of area A2 . E 3 × 10 −10
−1 −1
The ratio A 1 / A2 is = 2.5 × 10 m V s6 2

[NEET (Oct.) 2020] Hence, correct option is (a).


(a) 1.5 (b) 1 (c) 0.8 (d) 2
Ans. (b) 07 The color code of a resistance is O T(K)
given below [NEET (Sep.) 2020]
Given, R 1 = R 2 , l 1 = l 2
l For copper at 0K, value of resistivity is
Since, resistance, R = ρ⋅ 1.7 × 10 −8 Ω-m.
A
Hence, correct option is (b).
R 1 l 1 A2 R l A
⇒ = ⋅ ⇒ 1 = 1⋅ 2 Yellow Violet Brown Gold
R 2 l 2 A1 R 1 l 1 A1 09 Which of the following acts as a
A2 The values of resistance and circuit protection device?
⇒ 1= [QR 1 = R 2 ]
A1 tolerance respectively, are [NEET (National) 2019]
A1 (a) 47 k Ω, 10% (b) 4.7 k Ω, 5% (a) Inductor (b) Switch
⇒ =1 (c) 470 Ω, 5% (d) 470 k Ω, 5%
A2 (c) Fuse (d) Conductor
Ans. (c) Ans. (c)
05 The equivalent resistance According to the carbon colour code for Among given devices fuse is used in
resistors, electric circuit as a protective device.
between A and B for the mesh
Code of yellow = 4
shown in the figure is It helps in preventing excessive amount
Code of violet = 7 of current to flow in the circuit or from
[NEET (Oct.) 2020]
Code of brown, i.e. multiplier = 10 1 short circuiting. It has low melting point
Code of gold, i.e. tolerance = ± 5% and low resistivity, so when excess
4Ω 8Ω Hence, resistance of resistor amount of current flow in the circuit, it
= 47 × 10 1 Ω , 5% melts and break the circuit.

4Ω
6Ω = 470 Ω , 5%
8Ω 10 In the circuits shown below, the
Hence, correct option is (c).
readings of voltmeters and the
A B 08 Which of the following graph ammeters will be
[NEET (National) 2019]
represents the variation of
(a) 7.2 Ω (b) 16 Ω (c) 30 Ω (d) 4.8 Ω
resistivity (ρ) with temperature (T ) 10 Ω i1
Ans. (b)
for copper? [NEET (Sep.) 2020]
Equivalent resistance between points A
and B is given as ρ
V1 A1
(a)
4Ω 8Ω

T 10 V
4Ω 6Ω
8Ω ρ 10 Ω i2

(b)
A B 10 Ω
V2 A2
R AB = [(4 + 8) | | 6] + 4 + 8 = [12 | | 6] + 12 T
 12 × 6 
= ρ
 + 12 = [4] + 12 = 16Ω
 12 + 6  10 V
(c)
06 A charged particle having drift (a) V1 = V2 and i 1 > i 2 (b) V1 = V2 and i i = i 2
velocity of 7.5 × 10 −4 ms −1 in an T (c) V2 > V1 and i 1 > i 2 (d) V2 > V1 and i 1 = i 2
electric field of 3 × 10 −10 Vm −1 , has ρ Ans. (b)
a mobility (in m 2 V −1 s −1 ) of For an ideal voltmeter, the resistance is
[NEET (Sep.) 2020] (d) infinite and for an ideal ammeter, the
(a) 2.5 × 106 (b) 2.5 × 10−6 resistance is zero.
(c) 2.25 × 10−15 (d) 2.25 × 1015 T
10 V i1 12 The resistance of a wire is R ohm. If 14 Across a metallic conductor of
it is melted and stretched to n non-uniform cross-section, a
V1
times its original length, its new constant potential difference is
A1 resistance will be [NEET 2017] applied. The quantity which
R R remain constant along the
(a) nR (b) (c) n2 R (d)
n n2 conductor is [CBSE AIPMT 2015]
10 V Circuit 1 Ans. (c) (a) current density (b) current
So, the current in circuit 1 is Thinking Process Volume of material (c) drift velocity (d) electric field
remains same in stretching.
R × i = V or 10 i 1 = 10 Ans. (b)
10 As volume remains same, A1l 1 = A2 l 2
⇒ i1 = = 1A As the cross-sectional area of the
10 Now, givenl 2 = nl 1 conductor is non-uniform so current
∴ V1 = i 1 × R = 1 × 10 = 10 V Al A
∴New area A2 = 1 1 = 1 density will be different.
Similarly, for circuit 2, the addition of l2 n As I = JA …(i)
10 Ω to voltmeter does not affect the Resistance of wire after stretching It is clear from Eq. (i), when area
current and hence l nl
R 2 = ρ 2 = ρ⋅ 1 increases the current density decreases
10 Ω i2 A2 A1 / n so the number of flow of electrons will

10 Ω  l    l1   be same and thus the current will be


=  ρ 1  ⋅ n2 = n2 ⋅ R QR =  ρ  
V2  A1   A1   constant.
A2 
15 Two metal wires of identical
13 A, B and C are voltmeters of
dimensions are connected in
Circuit 2 resistance R, 1.5 R and 3R
10 V series. If σ 1 and σ 2 are the
respectively as shown in the
10 i 2 = 10 ⇒ i 2 =
10
= 1A conductivities of the metal wires
figure. When some potential
10 respectively, the effective
difference is applied between X
V2 = i 2 R = 1 × 10 = 10 V conductivity of the combination is
and Y, the voltmeter readings are
∴ V1 = V2 and i 1 = i 2 [CBSE AIPMT 2015]
VA , VB and VC respectively. Then, 2 σ1 σ2 σ1 + σ2
[CBSE AIPMT 2015] (a) (b)
11 A carbon resistor of (47 ± 4.7) kΩ σ1 + σ2 2 σ1 σ2
is to be marked with rings of
B σ + σ2 σ σ
A (c) 1 (d) 1 2
different colours for its X Y σ1 σ2 σ1 + σ2
C
identification. The colour code
Ans. (a)
sequence will be [NEET 2018] (a) VA = VB = VC (b) VA ≠ VB = VC
Net resistance of a metal wire having
(a) Yellow - Green - Violet - Gold (c) VA = VB ≠ VC (d) VA ≠ VB ≠ VC resistivity ρ, we have
(b) Yellow - Violet - Orange - Silver L L
Ans. (a)
(c) Violet - Yellow - Orange - Silver A
The equivalent resistance between Q
(d) Green - Orange - Violet - Gold ρ1 ρ2
and S is given by
Ans. (b) L
1.5 R R 1 = ρ1
Given, R = (47 ± 4.7) kΩ B A
= 47 × 10 3 ± 10% Ω RΩ L
P Q S Similarly, R 2 = ρ2
A A
As per the colour code for carbon
resistors, the colour assigned to Then, net effective resistance of two
C
numbers. 3R metal wires,
4 – Yellow 2L L L
Req = R 1 + R 2 ⇒ ρ = ρ1 + ρ2
7 – Violet A A A
3 – Orange V ⇒ 2ρ = ρ1 + ρ2
For ±10% accuracy, the colour is silver. 1
As, conductivity σ = , we have
Hence, the bands of colours on carbon ρ
resistor in sequence are yellow, violet, 1 1 1 2+ 1 2 1 1 2 σ1 + σ2
= + = = + ⇒ =
orange and silver. R ′ 1.5R 3R 3R σ σ1 σ2 σ σ1 σ2
Note To remember the colour code
⇒ R′ = R
sequence for carbon resistor, the ⇒Net effective conductivity of
following sentence should be kept in Now, VPQ = VA = IR
combined wires,
memory. B B Roy of Great Britain has a Also VQS = VB = VC = IR 2σ 1 σ 2
σ=
Very Good Wife. Hence, VA = VB = VC σ1 + σ2
16 A circuit contains an ammeter, a 19 A wire of resistance 12 Ω m −1 is bent Ratio of resistance before and after
to form a complete circle of radius 10 streching,
battery of 30 V and a resistance 2
40.8 Ω all connected in series. If cm. The resistance between its two R2 l2A1  l 2 
= ×
= 
the ammeter has a coil of diametrically opposite points A and B R 1 l 1 A2  l 1 
resistance 480 Ω and a shunt of as shown in the figure is R 2 (l + 10%l ) 2
=
20 Ω, then reading in the ammeter [CBSE AIPMT 2009] R1 l2
will be [CBSE AIPMT 2015]
 11 l 
2

(a) 0.5 A (b) 0.25 A R 2  10 


(c) 2 A (d) 1 A =
A B R1 l2
Ans. (a) R 2 = 1.21R 1
Effective resistance of a circuit,
RS=20 Ω 21 When a wire of uniform
(a) 0.6 π Ω (b) 3 Ω cross-section a, length l and
(c) 6 π Ω (d) 6 Ω resistance R is bent into a
40.8 Ω Ans. (a) complete circle, resistance
RA=480 Ω between two of diametrically
A B opposite points will be
[CBSE AIPMT 2005]
12 π Ω R R R
30 V 10 (a) (b) (c) 4R (d)
480 × 20 4 8 2
R eff = 40.8 +
480 + 20 Ans. (a)
⇒A B
= 40.8 + 192
. = 60 Ω When wire is bent to form a complete
circle, then
So, current flowing across ammeter,
12 π
V 30 1 Ω Total circumference,
I= = = = 0.5 A 10
2πr = Total length, R
R 60 2 10
Circumference of circle = 2 πr = 2 × π ⇒ r=
R
Hence, reading of ammeter = 0.5 A 100 2π
2π π
= = Resistance of each semicircle = πr
17 A wire of resistance 4Ω is 10 5 πR R
π 12 π = =
stretched to twice its original Resistance of wire = 12 × = 2π 2
length. The resistance of stretched 5 5
12 π
wire would be [NEET 2013] Resistance of each section = Ω
10
(a) 2 Ω (b) 4 Ω (c) 8 Ω (d) 16 Ω ∴ Equivalent resistance A B
Ans. (d) 12 π 12 π
×
As the resistance of stretched wire to = 10 10
length n times of original length is 12 π 12 π
+
R ′ = n2 R = 22 × 4 = 4 × 4 = 16 Ω 10 10 Thus, net resistance in parallel
where, R = original resistance 6π combination of two semicircular
⇒ = = 0.6 π Ω
R ′ = final resistance 10 resistances
R R R2
×
18 The mean free path of electrons in 20 A wire of a certain material is R′ = 2 2 = 4 =
R
a metal is 4 × 10 −8 m. The electric stretched slowly by 10 percent. Its R R
+ R 4
field which can give on an average new resistance and specific 2 2
2 eV energy to an electron in the resistance become respectively
metal will be in unit of Vm −1 [CBSE AIPMT 2008] 22 n resistances each of r ohm, when
[CBSE AIPMT 2009] (a) 1.2 times, 1.1 times connected in parallel give an
(a) 8 × 107 (b) 5 × 10−11 (b) 1.21 times, same equivalent resistance of R ohm. If
(c) 8 × 10−11 (d) 5 × 107 (c) Both remain the same these resistances were connected
Ans. (d) (d) 1.1 times, 1.1 times in series, the combination would
Energy = 2eV Ans. (b)
have a resistance in ohms, equal
eV0 = 2 eV ⇒ V0 = 2 After stretching, specific resistance (ρ)
to [CBSE AIPMT 2004]
2 R
So, electric field, E = will remain same. (a) n2 R (b)
4 × 10 −8 ρl n2
Original resistance of wire, R = R
= 0.5 × 10 8 A (c) (d) nR
= 5 × 10 7 V/m n
Ans. (a) Ans. (a) In V-I graph, for portion CD of graph, the
Equivalent resistance of n resistances The specific resistance (resistivity) of a current decreases with increasing
each of r ohmin parallel is given by metallic conductor nearly increases voltage.
1 1 1 n with increasing temperature as shown Thus, portionCD corresponds to
= + + ... + n times = negative resistance.
R r r r in figure. This is because, with the
increase in temperature the ions of the Alternative
So, r = nR
conductors vibrate with greater We know that the slope of graphV-I will
When these resistances are connected amplitude and the collisions between give the resistance, since slope of
in series, effective resistance is electrons and ions become more CD curve is negative.
R ′ = r + r + ... + n times = nr frequent, over a small temperature So, it has negative resistance.
∴ R ′ = n (nR) = n2 R range (upto 100°C). The resistivity of a
metal can be represented
27 There are three copper wires of
23 The electric resistance of a approximately by the equation
ρt = ρ0 (1 + αt)
length and cross-sectional area
certain wire of iron isR. If its
ρ
(L, A), (2 L, A /2)
length and radius are both
doubled, then [CBSE AIPMT 2004] (L/2, 2 A). In which case is the
(a) the resistance will be doubled and resistance minimum ?
the specific resistance will be halved [CBSE AIPMT 1997]
(b) the resistance will be halved and (a) It is the same in all three cases
the specific resistance will remain (b) Wire of cross-sectional area 2 A
unchanged (c) Wire of cross-sectional area A
t
(c) the resistance will be halved and 1
the specific resistance will be The factor α is called the temperature
(d) Wire of cross-sectional area A
2
doubled coefficient of resistivity.
(d) the resistance and the specific Ans. (b)
resistance will both remain 25 The resistance of a discharge tube The relation between length and area is
unchanged ρl
is [CBSE AIPMT 1999] R= ...(i)
A
Ans. (b) (a) zero (b) ohmic
The formula for resistance of wire is (c) non-ohmic (d) infinity where,ρbeing specific resistance is the
ρl proportionality constant and depends
R= Ans. (c)
A on nature of material.
In discharge tube, the current is due to L
where,ρ = specific resistance of the wire (i) Length = , area = 2 A
l flow of positive ions and electrons. 2
⇒ R∝ Moreover, secondary emission of
A Putting in Eq. (i), we have
l electrons is also possible. So, V-I curve ρ(L/2) ρL
⇒ R∝ 2 (QA = πr 2 ) is non-linear, hence its resistance is R= =
r 2A 4A
2 non-ohmic.
R 1 l 1 r2 (ii) Length = L, area = A
∴ = × 2 ...(i)
R 2 l 2 r1 26 From the graph between current I Putting in Eq. (i), we have
Given, l 1 = l , l 2 = 2l , r1 = r, r2 = 2 r, R 1 = R. ρL
and voltage V shown in figure, R=
Substituting these values in Eq. (i), we A
identify the portion corresponding A
have to negative resistance. (iii) Length = 2L, area =
R1 l (2 r) 2 2
= × 2 [CBSE AIPMT 1997]
R2 2l r Putting in Eq. (i), we have
I 2 L 4 ρL
R1 l (2 r) 2 E R =ρ =
= × 2 C A/2 A
R2 2l r
As it is understood from above,
R1 R D
= 2 ⇒ R2 = resistance is minimum only in option (b).
R2 2 B
Therefore, resistance will be halved. 28 What will be the equivalent
Now, the specific resistance of the wire A V resistance of circuit shown in
does not depend on the geometry of the figure between two points A and
(a) DE (b) CD
wire. Hence, it remains unchanged.
(c) BC (d) AB D? [CBSE AIPMT 1996]
24 The specific resistance of a Ans. (b) 10 Ω 10 Ω 10 Ω
conductor increases with ∆V A B
From Ohm’s law, resistance R =
[CBSE AIPMT 2002] ∆I
10 Ω 10 Ω
(a) increase in temperature Resistance is said to be negative if on
(b) increase in cross-sectional area increasing voltage or temperature, C D
(c) decrease in length current decreases. 10 W 10 Ω 10 Ω
(d) decrease in cross-sectional area (a) 10 Ω (b) 20 Ω (c) 30 Ω (d) 40 Ω
Ans. (c) per unit original resistance per degree Ans. (c)
10Ω rise of temperature and is given by For the same length and same material,
10Ω 10Ω
A B R − Ro R 2 A1 3
R1 R2 α= t or R t = R o (1 + α t) = =
Ro × t R 1 A2 1
R3 10Ω R2 10Ω
R5 R6 R t = Resistance at final temperature or R2 = 3 R1
C D
10Ω 10Ω 10Ω R o = Resistance at initial temperature The resistance of thick wire,
t = Change in temperature R 1 = 10 Ω
Effective resistance of R 2 and R 4 in Case I 5 = R o [1 + α (50)] …(i) The resistance of thin wire
series, Case II 7 = Ro [1 + α (100)] …(ii) = 3 R 1 = 3 × 10 = 30 Ω
R ′ = 10 + 10 = 20 Ω Dividing Eq. (i) by Eq. (ii) Total resistance = 10 + 30 = 40 Ω
Effective resistance of R 3 and R 5 in 5 1 + 50 α
=
series, 7 1 + 100 α 33 Three resistances each of 4 Ω are
R ′ ′ = 10 + 10 = 20 Ω connected to form a triangle. The
∴ 5 + 500 α = 7 + 350 α
Net total resistance of R ′ and R ′′ in 2 resistance between any two
∴ α= = 0.01/°C
parallel is 150 terminals is [CBSE AIPMT 1993]
20 × 20 (a) 12 Ω (b) 2 Ω
Rp = = 10 Ω
20 + 20 31 If a wire of resistance R is melted 8
(c) 6 Ω (d) Ω
∴ Total resistance between A and D and recasted to half of its length, 3
= 10 + 10 + 10 then the new resistance of the Ans. (d)
A
= 30 Ω wire will be [CBSE AIPMT 1995]
R R
(a) (b)
29 If a neglegibly small current is 4 2
4Ω 4Ω
passed through a wire of length 15 (c) R (d) 2 R
m and of resistance 5 Ω having Ans. (a)
uniform cross-section of Let initial resistance of wire be R, its C
B
6 × 10 −7 m2 , then coefficient of initial length isl 1 and final length isl 2 . 4Ω
According to probleml 2 = 0.5 l 1 , volume
resistivity of material, is Between any two terminals, two
of the wire is = lA. resistors of two arms are in series i.e.
[CBSE AIPMT 1996]
Since, the volume of wire remains the between B and C, equivalent resistance
(a) 1 × 10−7 Ω-m (b) 2 × 10−7 Ω-m
same after recasting, therefore is
(c) 3 × 10−7 Ω-m (d) 4 × 10−7 Ω-m l 1 A1 = l 2 A2 1 1 1
= +
Ans. (b) l 1 A2 l1 A R BC 4 8
∴ = or = 2
Resistance of a given conducting wire l 2 A1 0.5 l 1 A1 1 2+ 1
=
is given by A2 R BC 8
l ∴ =2
R = ρ⋅ 8
A
A1 ∴ R BC = Ω
l 3
where,ρis the specific resistance of the As resistance of wire, R ∝
A
material of the conductor. R 1 l 1 A2 l 34 The velocity of charge carriers of
Here, l = 15 m ∴ = × = 1 ×2=4
R 2 l 2 A1 0.5 l 1 current (about 1 A) in a metal
A = 6 × 10 −7 m2 R under normal conditions is of the
R
R = 5 Ω, ρ = ? or R2 = 1 =
RA 4 4 order of [CBSE AIPMT 1991]
∴ ρ= Note (a) a fraction of mm/s
l
5 × 6 × 10 −7 As we know the resistance R of wire of (b) velocity of light
= length l and area of cross-section, A is (c) several thousand m/s
15
given by R = ∫ l / A (d) a few hundred m/s
= 2 × 10 −7 Ω-m
Ans. (a)
30 If the resistance of a conductor is 32 Two wires of the same metal have The charged particles whose flow in a
5 Ω at 50°C and 7 Ω at 100°C, then same length, but their definite direction constitutes the
the mean temperature coefficient cross-sections are in the ratio 3 : 1. electric current are called current
carriers. In metals the valence
of resistance (of the material) is They are joined in series. The
electrons of the atoms do not remain
[CBSE AIPMT 1996] resistance of thicker wire is 10 Ω. attached to individual atoms but are
(a) 0.01/°C (b) 0.04/°C The total resistance of the free to move throughout the volume of
(c) 0.06/°C (d) 0.08/°C. combination will be the conductor. Their velocity under
Ans. (a) [CBSE AIPMT 1995] normal conditions is of the order of a
Temperature coefficient of resistance (a) 10 Ω (b) 20 Ω fraction of mm/s.
is defined as the increase in resistance (c) 40 Ω (d) 100 Ω
35 You are given several identical arm ED,therefore the resistance of Ans. (d)
network will not change, if the wiresCED
resistances each of value R = 10 Ω Let A1 , A2 , A3 be the area of
and AEB are disconnected at E,as shown cross-section of three wires of copper
and each capable of carrying a in Fig. (b). of masses m1 , m2 , m3 and lengthl 1 , l 2 , l 3
maximum current of 1A. It is (i 2 – i 3 ) respectively. Given m1 = m, m2 = 3 m,
required to make a suitable C D
i3 i2 m3 = 5m, l 1 = 5l , l 2 = 3l , l 3 = l
combination of these resistances r
r Mass = volume × density,
of 5 Ω which can carry a current of r
r
r
So, m = A1 × 5 l × ρ …(i)
4 A. The minimum number of i2 r
i3
r 3 m = A2 × 3 l × ρ …(ii)
resistances of the type R that will E i1 5 m = A3 × l × ρ …(iii)
(i 1 + i 2 ) A i 1 B (i1 + i2)
be required for this job is (a) From Eqs. (i) and (ii), we get
[CBSE AIPMT 1990]
Now, resistance of path AEB = r + r = 2 r A2 = 5 A1
(a) 4 (b) 10
Resistance of path From Eqs. (i) and (iii),
(c) 8 (d) 20
(2 r) × r 8r A3 = 25A1
ACDB = r + + r=
Ans. (c) (2 r) + r 3 l 5l
r ∴ R1 = ρ 1 = ρ
The paths AEB and ACDB are in parallel, A1 A1
1A therefore the effective resistance l2 3l 3
r between A and B will be R2 = ρ =ρ× = R1
1A A2 5 A1 25
1 1 3 4+ 3 7
1A r = + = = l ρ×l R
R 2 r 8r 8r 8r R3 = ρ 3 = = 1
1A r 8r A3 25 A1 125
or R=
7 3 R
∴ R1 : R2 : R3 = R1 : R1 : 1
To carry a current of 4 A we need four But r = 1 Ω, 25 125
path, each carrying a current of 1A. Let r 8×1 8
Therefore, R = = Ω = 125 : 15 : 1
be the resistance of each path. These 7 7
resistances are connected in parallel.
So, their equivalent resistance. 37 n equal resistors are first TOPIC 2
1 1 1 1 1 r connected in series and then
= + + + or rp = ⋅
rp r r r r 4 connected in parallel. What is the Heating Effect of Current
r ratio of the maximum to the
But, rp = = 5 (given)
4 minimum resistance ? 39 Six similar bulbs are connected as
[CBSE AIPMT 1989]
∴ r = 20 Ω 2 2
shown in the figure with a DC
(a) n (b) 1 /n (c) n (d) 1/n
For this purpose two resistances source of emf E and zero internal
should be connected is series. There Ans. (c) resistance.
are four such combinations in parallel. When resistors are connected in series, The ratio of power consumption
Hence, the total number of resistances then effective resistance of series
= 4 × 2 = 8. by the bulbs when (i) all are
combination
R S = R + R + K + n terms
glowing and (ii) in the situation
36 In the network shown in figure = nR …(i) when two from section A and one
each resistance is 1 Ω. The When resistors are connected in from section B are glowing, will be
effective resistance between A and parallel, then effective resistance [NEET (National) 2019]
B is [CBSE AIPMT 1990] 1 1 1
= + + K + n terms A B
1Ω RP R R
1 n
or = …(ii)
1Ω RP R
1Ω 1Ω
1Ω Rp =
R
1Ω 1Ω n
A B From Eqs. (i) and (ii), we get
R S n2
4 3 =
(a) Ω (b) Ω RP 1
3 2 E
8
(c) 7 Ω (d) Ω 38 The masses of the three wires of
7 (a) 9 : 4 (b) 1 : 2
copper are in the ratio of 1 : 3 : 5
Ans. (d) (c) 2 : 1 (d) 4 : 9
and their lengths are in the ratio of
The distribution of currents in the Ans. (a)
5 : 3 : 1. The ratio of their electrical
circuit is shown in the Fig. (a). Due to resistance is [CBSE AIPMT 1988] Case I When all bulbs are glowing, then
symmetry, current in arm AE is equal to the circuit can be realised as shown in
(a) 1 : 3 :5 (b) 5 : 3 :1
current in the arm EB.Since, current in the figure below.
the armCE is equal to the current in the (c) 1 :25 :125 (d) 125 :15 :1
A B 40 The charge following through a E 230
R R ∴ Current, i = =
resistance R varies with time t as Rnet R + R b
Q = at − bt 2 , where a and b are 230
R R ⇒ R + 20 = = 46
positive constants. The total heat 5
produced in R is [NEET 2016] ∴ R = 26 Ω
R R
a 3R a 3R a 3R a 3R
(a) (b) (c) (d)
3b 2b b 6b 42 Two cities are 150 km apart.
Ans. (d)
Electric power is sent from one
E city to another city through
Given, charge Q = at − bt 2 …(i)

copper wires. The fall of potential
∴The equivalent resistance of this Q We know that current, I = per km is 8 V and the average
circuit is dt
So, eq. (i) can be written as resistance per km is 0. 5 Ω. The
R eq = R A + R B
d power loss in the wire is
As, section A has three parallel I = (at − bt 2 ) ⇒ I = a − 2bt …(ii)
dt [CBSE AIPMT 2014]
resistance, so equivalent resistance,
R For maximum value of t, till the current (a) 19.2 W (b) 19.2 kW
RA = exist is given by (c) 19.2 J (d) 12.2 kW
3
Similarly, for section B, equivalent ⇒ a − 2bt = 0 Ans. (b)
a
resistance, ∴ t= …(iii) Potential drop between two cities is
R 2b = 150 × 8 = 1200 V
RB =
3 Q The total heat produced (H) can be Average resistance of total wire

R R 2R
R eq = + = … (i) given as = 0.5 × 150 = 75 Ω
3 3 3 t V2
H = ∫ I 2 R dt So, power loss P =
Thus, power consumed in this circuit, 0 R
V 2 E 2 3E 2
a /2 b 1200 × 1200
P1 = = = [using Eq. (i)] =∫ (a − 2bt) 2 R . dt = = 19200 W
R R eq 2R
0
75
Q t = a  = 19.2 kW
…(ii)  
 2b 
Case II When two from section A and
=∫
a /2 b
(a 2 + 4b 2 t 2 − 4abt) Rdt 43 If voltage across a bulb rated 220
one from section B glow, the circuit can
be realised as shown in the figure
0
V-100 W drops by 2.5% of its rated
a /2 b
below.  t 3 4abt 2  value, the percentage of the rated
H = a 2 t + 4b 2 −  R
 3 2 0 value by which the power would
A B
R decrease is [CBSE AIPMT 2012]
R Solving above equation, we get
R
(a) 20% (b) 2.5%
a 3R
⇒ H= (c) 5% (d) 10%
6b
Ans. (c)
V2
41 A filament bulb (500 W, 100 V) is to Power, (P) =
E be used in a 230 V main supply. R
∴Equivalent resistance of section A, When a resistance R is connected For small variation,
R ∆P ∆V
RA = in series, it works perfectly and × 100 = 2 × × 100 = 2 × 2.5 = 5%
2 P V
the bulb consumes 500 W. The
and of section B, Therefore, power would decrease by
value of R is [NEET 2016]
5%.
RB = R
(a) 230 Ω (b) 46 Ω
Thus, equivalent resistance of the (c) 26 Ω (d) 13 Ω
entire circuit becomes 44 If power dissipated in the 9 Ω
R 3R Ans. (c) resistor in the circuit shown is
R eq = R A + R B = + R = … (iii)
2 2 If a rated voltage and power are given, 36W, the potential difference
∴Power consumed by this circuit, then
V2
Prated = rated across the 2 Ω resistor is
V 2 E 2 2E 2 R [CBSE AIPMT 2011]
P2 = = = P 9Ω
R R eq 3R ∴ Current in the bulb, i = (Q P = Vi)
V
[using Eq. (iii)]… (iv) 500 6Ω
So, ratio of power of two cases is i= =5A
100
obtained from Eqs. (ii) and (iv), we get
P1 3E 2 ∴ Resistance of bulb,
3R 9
= × = or 9 :4 100 × 100 + –
P2 2R 2E 2 4 Rb = = 20 Ω V 2Ω
500
Q Resistance R is connected in series. (a) 8 V (b) 10 V (c) 2 V (d) 4 V
Ans. (b) 47 The total power dissipated in watts So, from Kirchhoff’s law, V1 = V2
Electric power, P = i 2 R in the circuit shown here is R ′ × i 1 = 8i 2 or 4 × i 1 = 8i 2
P [CBSE AIPMT 2007] 8
∴ Current, i = or i1 = i2 = 2i2
R 6Ω 4
For resistance of 9 Ω, or i1 = 2i2 ...(i)
36 Power dissipated across 8 Ω resistance
i1 = = 4 = 2A
9 is
i × R 2×9 3Ω i 22 (8) t = 2 W [QP = iRt]
i2 = 1 = =3A 2
6 6 or i 2 t = = 0.25 W
2
...(ii)
I = i1 + i2 = 2 + 3 = 5 A – 8
4Ω +
So, V2 = IR 2 = 5 × 2 = 10 V 18 V Power dissipated across3 Ω resistance
is
(a) 16 (b) 40 (c) 54 (d) 4
45 An electric kettle takes 4 A H = i 12 (3) t
current at 220 V. How much time Ans. (c) = (2 i 2 ) 2 (3) t = 12 i 22 t
will it take to boil 1 kg of water The resistance of6 Ω and 3 Ω are in but i 2 t = 0.25 W
2

parallel in the given circuit, their H = 12 × 025


. =3W
from temperature 20°C? equivalent resistance is
[CBSE AIPMT 2008] 1 1 1 1+ 2 1
= + = = or R 1 = 2 Ω 49 A 5 A fuse wire can withstand a
(a) 6.3 min (b) 8.4 min R1 6 3 6 2
(c) 12.6 min maximum power of 1 W in circuit.
Again, R 1 is in series with4Ω The resistance of the fuse wire is
(d) 4.2 min
resistance, hence [CBSE AIPMT 2005]
Ans. (a)
R = R1 + 4 = 2 + 4 = 6 Ω (a) 0.2 Ω (b) 5 Ω
Vit = ms ∆t Thus, the total power dissipated in the
ms ∆ t (c) 0.4 Ω (d) 0.04 Ω
⇒ t= circuit Ans. (d)
Vi V2
1 × 4200 × 80 P= ; Here, V = 18 V, R = 6 Ω As we know that,
= R
220 × 4 Power P = i 2 R
(18) 2
Thus, P= = 54 W  where, i = current in circuit
= 381.82 s 6  
 R = resistance 
= 6.3 min
48 Power dissipated across the 8 Ω p
R= 2
46 A current of 3 A flows through the resistor in the circuit shown here i
2 Ω resistor shown in the circuit. is 2 W. The power dissipated in Given, P = 1 W, i = 5 A
The power dissipated in the 5 Ω watt units across the 3 Ω resistor R=
1
= 0.04 Ω
resistor is [CBSE AIPMT 2008] is [CBSE AIPMT 2006] (5) 2
2Ω 1Ω 3Ω i1
50 In India, electricity is supplied for
4Ω
i i domestic use at 220 V. It is
supplied at 110 V in USA. If the
resistance of a 60 W bulb for use
8Ω i2
in India is R, the resistance of a 60
1Ω 5Ω W bulb for use in USA will be
(a) 2.0 (b) 1.0 (c) 0.5 (d) 3.0
[CBSE AIPMT 2004]
Ans. (d) R R
Problem Solving Strategy Find the (a) R (b) 2 R (c) (d)
(a) 4 W (b) 2 W 4 2
(c) 1 W (d) 5 W equivalent resistance of the circuit and
then calculate the power dissipated by Ans. (c)
Ans. (d)
the circuit. Since, power rating of bulb in both the
Voltage across 2 Ω, Resistances 1Ω and 3 Ω are connected countries India and USA is same, so
=3×2=6V in series, so, effective resistance V12 V2
2
 V2 
=  P=
Voltage across 4 Ω and (5 Ω + 1 Ω) R′ = 1 + 3 = 4 Ω R1 R2  R 
resistor is same. Now, R ′ and 8 Ω are in parallel. We know
6 220 × 220 110 × 110
So, current across 5 Ω = = 1A that potential difference across ⇒ =
1+ 5 resistances in parallel is same. R1 R2
 V 1Ω 3Ω R2 110 × 110
i= i1 ⇒ =
 R  R 1 220 × 220
Power across 5 Ω = P = i 2 R i i
⇒ R2 =
R
(QR 1 = R)
= (1) 2 × 5 4
=5W
8Ω i2
51 When three identical bulbs of 60 220 V 200 V 53 An electric kettle has two heating
W, 200 V rating are connected in 100 W 100 W coils. When one of the coils is
series to a 200 V supply, the connected to an AC source, the
power drawn by them will be water in the kettle boils in 10 min.
[CBSE AIPMT 2004] When the other coil is used the
(a) 60 W (b) 180 W (c) 10 W (d) 20 W water boils in 40 min. If both the
Ans. (d) 220 V coils are connected in parallel, the
Let R 1 , R 2 and R 3 are the resistances of time taken by the same quantity of
In series, R eq = R 1 + R 2
three bulbs respectively. water to boil will be
220 × 220 
In series order, = 
[CBSE AIPMT 2003]
 ×2
R = R1 + R2 + R3  100  (a) 25 min (b) 15 min
V2 2
220 × 220 (c) 8 min (d) 4 min
but, R = and supply voltage in series Hence, Peq =
V
=
P Req  220 × 220 × 2 Ans. (c)
order is the same as the rated voltage.  
 100  Let R 1 and R 2 be the resistances of the
V2 V2 V2 V2
∴ = + + 100 coils, V be the supply voltage,H be the
P P1 P2 P3 = = 50 W heat required to boil the water.
2
1 1 1 1 V 2t1
or = + + Case II When two bulbs are connected For first coil, H = ...(i)
P 60 60 60 in parallel. R1
60 V 2t2
or P= = 20 W
3 220 V For second coil, H = ...(ii)
100 W R2
Alternative
Equating Eqs. (i) and (ii), we get
As three bulbs have same power and t
t1
voltage so, they have equal resistance. = 2
So, power equivalent when connected R1 R2
220 V
in series is given by, R 2 t 2 40 t = 10 min 
1 1 1 1 100 W i.e. = = =4  1 
= + + R 1 t 1 10 t 2 = 40 min
PEQ P1 P2 P3
1 1 1 1 ⇒ R 2 = 4R 1 ...(iii)
= + + (as P1 = P2 = P3 = P) When the two heating coils are in
PEQ P P P
parallel, equivalent resistance is given
P 60
So, PEQ = ⇒ = 20 W 220 V
by
3 3 R1R2 R × 4R 1 4R 1
R1 R2 R= = 1 =
In parallel, Req = R 1 + R 2 R 1 + 4R 1 5
52 Two 220 V, 100 W bulbs are R1 + R2
V 2t
connected first in series and then 2 and H= ...(iv)
 220 × 220  R
in parallel. Each time the  
 100  Comparing Eqs. (i) and (iv), we get
combination is connected to a =
220 × 220
×2 V 2t1 V 2t
220 V AC supply line. The power 100 =
R1 R
drawn by the combination in each 220 × 220 1
case respectively will be R eq = × ⇒ t=
R
× t1 [t 1 = 10 min]
100 2 R1
[CBSE AIPMT 2003]
V2 220 × 220 4
(a) 200 W, 150 W (b) 50 W, 200 W Hence, Peq = = ∴ t = × 10 = 8 min
R eq 220 × 220 × 1 5
(c) 50 W, 100 W (d) 100 W, 50 W
100 2
Ans. (b) = 200 W 54 Fuse wire is a wire of
Power P = 100 W, Voltage, V = 220 V Alternative [CBSE AIPMT 2003]
V2 P1 P2 (a) low resistance and low melting
P= For series, Peq =
R P1 + P2 point
V 2 (220) 2 100 × 100 (b) low resistance and high melting
∴ R= = = = 50 W
P 100 200 point
220 × 220
= Ω For parallel, Peq = P1 + P2
(c) high resistance and high melting
100 point
= 100 + 100 = 200 W
As both the bulb have same voltage and (d) high resistance and low melting
power so, resistance of bulbs will also Note Power equivalent of two or more point
be same. resistance in series is given by
1 1 1 Ans. (d)
Case I When two bulbs are connected = + and for parallel
in series. Peq P1 P2 The electric fuse is a device which is
used to limit the current in an electric
combination Peq = P1 + P2
circuit. Thus, the use of fuse
2
safeguards the circuit and the i  Ans. (d)
appliances connected in the circuit or ∆ θ2 =  2  ∆ θ1 ... (i)
 i1  V2
from being damaged. It is always Power consumed, P =
R
connected with the live (or phase) wire. Given, i 2 = 2 i 1 , ∆θ1 = 5° C V ′2  As resistance is same
The fuse wire is a short piece of wire So, from Eq. (i), and P ′ =  
made of a material of high resistance 2 R  for both the cases 
 2i 
and low melting point so that it may ∴ ∆θ2 =  1  × 5 Given, V = 200 V
easily melt due to overheating when  i1 
V ′ = 160 V, P = 100 W
excessive current passes through it. =4× 5
Comparing two different cases of
= 20 ° C voltage supplied
55 Two bulbs 25 W, 220 V and 100 W, 2
220 V are given. Which has higher P′  V ′ 
57. Three equal resistors connected So, = 
resistance? [CBSE AIPMT 2000] P V 
in series across a source of emf 2
(a) 25 W bulb together dissipate 10 W of power.  160 
=  = (0.8)
2

(b) 100 W bulb  200 


What will be the power dissipated
(c) Both bulbs will have equal
in watt if the same resistors are or P ′ = (0.8) 2 P
resistance
connected in parallel across the = 0.64 × 100 = 64 W
(d) Resistance of bulbs cannot be
compared same source of emf ? 59 A heating coil is labelled 100 W,
[CBSE AIPMT 1998]
Ans. (a) 10
220 V. The coil is cut in two equal
V2 (a) (b) 10 halves and the two pieces are
Power of electric bulb, P = 3
joined in parallel to the same
R (c) 30 (d) 90
V2 source. The energy now liberated
So, resistance of electric bulb, R = Ans. (d) per second is [CBSE AIPMT 1995]
P ∆U ∆q
Given, P1 = 25 W, P2 = 100 W, Power, P = =V = Vi (a) 25 J (b) 50 J
∆t ∆t
V1 = V2 = 220 V (c) 200 J (d) 400 J
Therefore, for same potential V2
or P = Vi = (QV = iR) Ans. (d)
difference V, R When heating coil is cut into two equal
1
R∝ When resistors are in series, then parts and these parts are joined in
P R1 = R + R + R parallel, then the resistance of the coil
Thus, we observe that for minimum 1
= 3R is reduced to of the previous value. As
power, resistance will be maximum and 4
vice-versa. ∴ Power dissipated,  1
 H ∝  , for constant voltage so,
Hence, resistance of 25 W bulb is V2 V2  R
maximum and 100 W bulb is minimum. P1 = = …(i)
R 1 3R energy liberated per second becomes 4
times, i.e.
When resistors are in parallel, then
56 A 5°C rise in temperature is 1 1 1 1 4 × 100 = 400 J.
observed in a conductor by = + +
R2 R R R
passing a current. When the 60 A 4µF conductor is charged to
3
current is doubled the rise in = 400 V and then its plates are
R
temperature will be approximately R joined through a resistance of 1kΩ.
[CBSE AIPMT 1998] ⇒ R2 = The heat produced in the
3
(a) 16°C (b) 10°C (c) 20°C (d) 12°C V2 V2 3V 2 resistance is [CBSE AIPMT 1994]
∴ P2 = = = …(ii)
Ans. (c) R 2 R /3 R (a) 0.16 J (b) 1.28 J
Energy loss in conductor, Q = i Rt 2 (c) 0.64 J (d) 0.32 J
By taking ratio of Eqs. (i) and (ii)
where,i = current flowing through it P2 3V 2 3R
Ans. (d)
R = resistance of conductor = × 2 =9 The energy stored in the capacitor
P1 R V
t = time for which current is 1
= CV 2 .
passed P2 = 9P1 2
Heat developed = ms ∆θ = 9 × 10 = 90 W
This energy will be converted into heat
∴ ms ∆θ = i 2 Rt
58. A 100 W 200 V bulb is connected in the resistor.
Since m, s , R, t remains constant. 1
So, ∆θ ∝i 2 to a 160 V power supply. The ∴ H = CV 2
2
So, for two different cases power consumption would be where,C = capacitance of capcitor
∆ θ2 i 22 [CBSE AIPMT 1997]
V = voltage across the plate of capacitor
=
∆ θ1 i 12 (a) 125 W (b) 100 W 1
(c) 80 W (d) 64 W ⇒ H = × 4 × 10 −6 × (400) 2 = 0.32 J
2
61 40 electric bulbs are connected in Ans. (b) Ans. (d)
series across a 220 V supply. After The circuit diagram is shown below The given circuit diagram can be drawn
one bulb is fused the remaining 39 as shown below
2V 1Ω 4V 1Ω
are connected again in I1 20Ω B 30Ω
series across the same supply. A E
The illumination will be V
[CBSE AIPMT 1989] 4Ω
I 30Ω 20Ω
(a) more with 40 bulbs than with 39 D F
I2 C
(b) more with 39 bulbs than with 40 I
(c) equal in both the cases Applying KVL in the loop, we get
(d) in the ratio 402 : 39 2 4I + I ⋅ 1 − 4 + I ⋅ 1 − 2 = 0
⇒ 6I = 6
Ans. (b) 2V
⇒ I = 1A
As the voltage is same for both forty The equivalent resistance of circuit is
and thirty nine bulbs combination, given by
therefore heat produced is given by
64 For the circuit given below, the
1 1 1
V 2t 1 Kirchhoff’s loop rule for the loop = +
H= and H ∝ . As equivalent R eq R AE R DF
R R BCDEB is given by the equation
1 1
resistance decreases, the combination [NEET (Oct.) 2020] = +
of 39 bulbs will glow more. (20 + 30) (30 + 20)
i1 R2 i2
A B C 1 1 2
= + =
62 A current of 2 A, passing through a 50 50 50
R1
conductor produces 80 J of heat E1 E2 ⇒ R eq = 25 Ω
in 10 s. The resistance of the V 2
i3 The current in circuit, I = = A
conductor in ohm is F E D R 25
[CBSE AIPMT 1989] E3
As the resistance of two branches is
(a) 0.5 (b) 2 same i.e. 50 Ω.
(c) 4 (d) 20 (a) − i 2 R2 + E2 − E3 + i 3 R1 = 0
So, the currentI 1 = I 2
Ans. (b) (b) i 2 R2 + E2 − E3 − i 3 R1 = 0
⇒ I = I1 + I2
(c) i 2 R2 + E2 + E3 + i 3 R1 = 0
Amount of heat produced in a 2 1
(d) −i 2 R2 + E2 + E3 + i 3 R1 = 0 = 2I 1 ⇒ I 2 = I 1 = A
conductor is equal to work done in 25 25
carrying a chargeq from one end of Ans. (b)
∴The voltage across AB
conductor to other end of conductor The circuit diagram is given below 1
having potential differenceV. V1 = I 1 R 1 = × 20
A I1 B
R2 I2 C
25
∴ H = W = Vq = Vit = i 2 Rt [as, V = IR]
and voltage across CD
∴ H = i 2 Rt J E1
R1
V2 = I 2 R 2 =
1
× 30
E2
H 25
⇒ R= 2 I3
(i t) ∴Voltmeter reading = V2 − V1
F E D 30 20 10
Given, H = 80 J, i = 2 A, t = 10 s, E3 = − = = 0.4 V
80 25 25 25
So, R = 2 =2Ω Applying KVL rule in loop BCDEB,
(2) × 10
R 2I2 + E 2 − E 3 − I3 R 1 = 0 66 A set of ‘n’ equal resistors, of value
TOPIC 3 65 The reading of an ideal voltmeter
‘R ’ each , are connected in series
to a battery of emf ‘E ’and internal
Cells and its Combination in the circuit shown is resistance ‘R ’. The current drawn
and Kirchhoff’s Rules [NEET (Odisha) 2019]
is I. Now, the ‘n’ resistors are
20 Ω 30 Ω
connected in parallel to the same
63 For the circuit shown in the figure, battery. Then, the current drawn
the current I will be V from battery becomes 10I. The
[NEET (Oct.) 2020] value of ‘n’ is [NEET 2018]
2V 1Ω 4V 1Ω 30 Ω 20 Ω (a) 20 (b) 11 (c) 10 (d) 9
Ans. (c)
When n equal resistors of resistance R
4Ω 2V are connected in series, then the
I
current drawn is given as
(a) 0.6 V (b) 0 V E
(a) 0.75 A (b) 1 A (c) 0.5 V (d) 0.4 V I=
(c) 1.5 A (d) 0.5 A nR + r
where, Equivalent internal resistance, 70 In the circuit shown, the cells A
nR = equivalent resistance of n req = nr and B have negligible resistances.
resistors in series and r = internal E nE For VA = 12 V, R 1 = 500 Ω and
∴ Current, I = eq =
resistance of battery. req nr R = 100 Ω the galvanometer (G)
Given, r = R E shows no deflection. The value of
or I = = constant
E E r VB is [CBSE AIPMT 2012]
⇒ I= = …(i)
nR + R R (n + 1) Thus, current (I) is independent of the R1
Similarly, when n equal resistors are number of cells (n) present in the circuit. G
connected in parallel, then the current Therefore, the graph showing the
drawn is given as relationship betweenI and n would be as
VA R VB
E shown below.
I′ =
R
+R
n I
R
where, = equivalent resistance of n (a) 4 V (b) 2 V
n
(c) 12 V (d) 6 V
resistors in parallel.
Ans. (b)
Given, I ′ = 10 I
E nE O n Concept If potential difference across
⇒ 10I = = …(ii) R Ω resistor is equal to potential
R
+R ( n + 1) R difference of cell B, galvanometer
n
68 The potential difference (VA − VB ) shows no deflection.
Substituting the value ofI from Eq. (i) in Applying Kirchhoff’s law,
Eq. (ii), we get between the points A and B in the
500I + 100I = 12
 E  nE given figure is [NEET 2016]
10  = 12 × 10 −2
So, I= = 2 × 10 −2 A
 R (n + 1)  R (n + 1) VA 2W
3V
1W VB 6
+ –
⇒ n = 10 Hence, VB = 100 (2 × 10 −2 ) = 2 V
AI = 2 A B
67 A battery consists of a variable (a) –3 V (b) +3 V 71 A current of 2 A flows through a 2
number ‘n ’ of identical cells (c) +6 V (d) +9 V Ω resistor when connected across
(having internal resistance ‘r ’each) Ans. (d) a battery. The same battery
which are connected in series. 2A 1 Ω + – 1Ω supplies a current of 0.5 A when
VA VB
The terminals of the battery are 3V
connected across a 9Ω resistor.
short-circuited and the current I is The internal resistance of the
Applying KVL,
measured. Which of the graphs battery is [CBSE AIPMT 2011]
VA + Σ V = VB + 2 × 2 + 2 × 1
shows the correct relationship (a) 1/3 Ω (b) 1/4 Ω
VA − VB − 3 = 4 + 2; VA − VB = 9 V
between I and n? [NEET 2018] (c) 1 Ω (d) 0.5 Ω
69 The internal resistance of a 2.1 V Ans. (a)
I I
cell which gives a current of 0.2 A Current in circuit connected with
battery of emf E with internal
(a) (b) through a resistance of 10 Ω is
resistance r is given by
[NEET 2013]
E
(a) 0.2 Ω (b) 0.5 Ω Current, i =
O n O n R+r
(c) 0.8 Ω (d) 1.0 Ω
E
I
Ans. (b) Case I 2= …(i)
I 2+ r
E
As, I =
R+r E
(c) (d) Case II 0.5 = …(ii)
E = emf of cell 9+ r
R = external resistance
O O From Eqs. (i) and (ii), we have,
n n r = internal resistance
2 9+ r
I = current in circuit =
Ans. (c) 0.5 2 + r
or E = I (R + r) ,
If n identical cells are connected in 9+ r
2.1 = 0.2 (10 + r) 4=
series, then 2.1 2+ r
10 + r = × 10
Equivalent emf of the combination, 2 ⇒ 3r = 1
E eq = nE ∴ r = 10.5 − 10 = 0.5 Ω 1
⇒ r= Ω
3
A
72 Consider the following two 75 Two cells, having the same emf
statements: are connected in series through 4Ω 4Ω
I. Kirchhoff’s junction law follows an external resistance R. Cells
from the conservation of charge. have internal resistances r1 and r2
II. Kirchhoff’s loop law follows (r1 > r2 ) respectively. When the
from the conservation of energy. circuit is closed, the potential
1Ω 3Ω
Which of the following is correct? difference across the first cell is
[CBSE AIPMT 2010] zero. The value of R is
B
(a) Both I and II are wrong [CBSE AIPMT 2006]
(b) I is correct and II is wrong r1 + r2
(a) r1 − r2 (b)
(c) I is wrong and II is correct 2 V
(d) Both I and II are correct r1 − r2
(c) (d) r1 + r2 (a) flow from A to B
Ans. (d) 2
(b) flow in the direction which will be
Kirchhoff’s junction law follows from Ans. (a) decided by the value of V
the conservation of charge. Net resistance of the circuit (c) be zero
Kirchhoff’s loop law follows from the = r1 + r2 + R (d) flow from B to A
conservation of energy. Net emf in series = E + E = 2 E
Ans. (d)
E E
73 See the electrical circuit shown in Resistances 4 Ω and 4 Ω are connected
this figure. Which of the following in series, so their effective resistance is
equations is a correct equation for R′ = 4 + 4 = 8Ω
it ? [CBSE AIPMT 2009] i r1 r2 r1 > r2 Similarly, 1Ω and 3 Ω are in series
R So, R′′ = 1 + 3 = 4 Ω
Now R ′ and R ′ ′ will be in parallel, hence
ε1 r1 effective resistance
i1
R R′ × R′′
R=
Therefore, from Ohm’s law, current in R′ + R′′
i2 the circuit 8 × 4 32 8
Net emf = = = Ω
r2 ε2 i= 8 + 4 12 3
Net resistance
(a) ε 1 − (i 1 + i 2 ) R − i 1 r1 = 0 2E Current through the circuit, from Ohm’s
⇒ i= ...(i)
(b) ε 2 − i 2 r2 − ε 1 − i 1 r1 = 0 r1 + r2 + R law
V 3V
(c) − ε 2 − (i 1 + i 2 ) R + i 2 r2 = 0 It is given that, as circuit is closed, i= = A
R 8
potential difference across the first cell
(d) ε 1 − (i 1 + i 2 ) R + i 1 r1 = 0 Let currents i 1 and i 2 flow in the
is zero.
Ans. (a) branches as shown. As voltage remains
i.e., V = E – ir1 = 0
The algebraic sum of the changes in E same in parallel combination
⇒ i= ...(ii) i.e. V1 = V2
potential in complete transversal of a r1
mesh (closed loop) is zero. i.e. ΣV = 0 A
Equating Eqs. (i) and (ii), we get
So, ε1 − (i 1 + i 2 ) R − i 1 r1 = 0
E 2E 4Ω
= ⇒ 2 r1 = r1 + r2 + R 4Ω
74 A student measures the terminal r1 r1 + r2 + R
potential difference (V ) of a cell
∴ R = external resistance i1
(of emf ε and internal resistance r) i2
⇒ R = r1 − r2
as a function of the current (I )
flowing through it. The slope and Note
1Ω 3Ω
The question is wrong as the statement
intercept of the graph between V i
is when the circuit is closed, the
and I, respectively, equal to potential difference across the first cell B
[CBSE AIPMT 2009] is zero which implies that in a series
(a) ε and − r (b) − r and ε circuit, one part cannot conduct current
V
(c) r and − ε (d) −ε and r which is wrong, Kirchhoff ’s law is
violated. The question must have been ∴ 8i 1 = 4 i 2
Ans. (b) modified. ⇒ i2 = 2 i1 ...(i)
According to Ohm’s law Also, i = i1 + i2
dV 76 In the circuit shown, if a 3V
= − r and V = ε if I = 0 ⇒ = i1 + 2 i1
dI conducting wire is connected 8
[As V + Ir = ε] between points A and B, the V V
⇒ i1 = A and i 2 = A
So, slope of the graph = − r and current in this wire will 8 4
intercept = ε [CBSE AIPMT 2006]
Potential drop at A, 78 A 6 V battery is connected to the 80 For a cell, the terminal potential
4V V
VA = 4 × i 1 = = terminals of a 3m long wire of difference is 2 .2 V when circuit is
8 2
uniform thickness and resistance open and reduces to 1.8 V when
Potential drop at B,
V V of 100 Ω. The difference of cell is connected to a resistance
VB = 1 × i 2 = 1 × = potential between two points on R = 5 Ω, the internal resistance (r)
4 4
Since, drop of potential is greater in4Ω the wire separated by a distance of cell is [CBSE AIPMT 2002]
resistance so, it will be at lower of 50 cm will be 10 9
(a) Ω (b) Ω
potential than B. Hence, on connecting [CBSE AIPMT 2004] 9 10
wire between points A and B the current (a) 2 V (b) 3 V 11 5
will flow from B to A. (c) Ω (d) Ω
(c) 1 V (d) 1.5 V 9 9
Ans. (c) Ans. (a)
77 Two batteries, one of emf 18V and
internal resistance 2 Ω and the Total current drawn from the battery In an open circuit, emf of cell
E 6 E = 2 .2 V
other of emf 12 V and internal i= = = 0.06 A
R + r 100 + 0 In a closed circuit, terminal potential
resistance 1 Ω, are connected as
Resistance of 50 cm wire is difference
shown. The voltmeter V will record
ρl ′  ρ  ρl V = 1.8 V
=   l ′ =   l ′ Q R = 
R
a reading of [CBSE AIPMT 2005] R′ =
A  A l   A External resistance, R = 5 Ω
100 Thus, internal resistance of cell is
= × 50
 2 .2 
r =  − 1 R = 
300 E
2Ω − 1 5
50 V   1.8 
So, R′ = Ω
18 V 3  11  2 10
=  − 1 5 = × 5 = Ω
1Ω Hence, the potential difference 9  9 9
12 V between two points on the wire
separated by a distancel ′ is given by 81 A cell has an emf 1.5 V. When
(a) 15 V (b) 30 V (c) 14 V (d) 18 V Ohm’s law connected across an external
Ans. (c) i.e. V = i R′ resistance of 2 Ω, the terminal
50
It is clear that the two cells oppose = 0.06 × = 1V potential difference falls to 1.0 V.
each other hence, the effective emf in 3
The internal resistance of the cell
closed circuit is 18 – 12 = 6 V and net is [CBSE AIPMT 2000]
resistance is 1 + 2 = 3 Ω (because in the 79 A battery is charged at a potential
closed circuit the internal resistances of 15 V for 8 h when the current (a) 2 Ω (b) 1.5 Ω
of two cells are in series). (c) 1.0 Ω (d) 0.5 Ω
flowing is 10 A. The battery on
The current in circuit will be in direction discharge supplies a current of 5 A Ans. (c)
of arrow shown in figure. Internal resistance of the cell is given by
for 15 h. The mean terminal
E −V 
V voltage during discharge is 14 V. r =  R
The watt-hour efficiency of the  V 
18 V
battery is [CBSE AIPMT 2004] Given, E = 1.5 V, V = 10
. V, R = 2 Ω
2Ω (a) 82.5% (b) 80%  1.5 − 10
.  0.5
(c) 90% (d) 87.5% ∴ r=  × 2= × 2= 1 Ω
 10 .  10
.
12 V Ans. (d)
1Ω Input energy when the battery is 82 The current (i) in the given circuit
charged is [CBSE AIPMT 1999]
Effective emf 6
I= = =2A E in = Vit
Total resistance 3 i
= 15 × 10 × 8 B
The potential difference across V will be = 1200 Wh
same as the terminal voltage of either
Energy released when the battery is RA = 3 Ω RB = 6 Ω
cell. 4.8 V
discharged
Since, current is drawn from the cell of
E out = 14 × 5 × 15
18 V, hence, C
= 1050 Wh A RC = 6 Ω
V1 = E 1 − ir1 = 18 − (2 × 2) = 18 − 4 = 14 V
Hence, watt hour efficiency of battery is
Similarly, current enters in the cell of 12 (a) 1.6 A (b) 2 A
given by
V, hence, (c) 0.32 A (d) 3.2 A
Energy output 1050
V2 = E 2 + ir2 = 12 + 2 × 1 = 12 + 2 = 14 V = = Ans. (b)
Energy input 1200
Hence, V = 14 V In the given circuit, resistances R B and
= 0.875 = 87.5%
R C are in series order, so their effective
resistance,
R ′ = RB + RC 86 Current through 3 Ω resistor is 0.8
= 6 + 6 = 12 Ω i3
i2
A, then potential drop through 4 Ω
Now, R A and R ′ are in parallel order, resistor is [CBSE AIPMT 1993]
hence net resistance of the circuit i1 3Ω
R ′ × R A 12 × 3 36 4Ω
R= = = Ω i
R ′ + R A 12 + 3 15 6Ω

The current flowing in the circuit,


or i = i1 + i2 + i3
V 15
i = = 4.8 × = 2A dQ dQ 1 dQ 2 dQ 3 + –
R 36 = + +
dt dt dt dt
(a) 9.6 V (b) 2.6 V
83 The current in the following circuit or, dQ = dQ 1 + dQ 2 + dQ 3 (c) 4.8 V (d) 1.2 V
is [CBSE AIPMT 1997] So, we can say that when a steady Ans. (c)
current flows through circuit, then
A 3Ω
neither accumulation of charges takes i1
place at the junction in the circuit nor 4Ω
i
2V any charge is removed from there.
3Ω 3Ω 6Ω
That’s why Kirchhoff ’s first law deals
with the conservation of charge. i2
C i
i B 3Ω
85 In the circuit of figure, the current + –
2
(a) 1 A (b) A in 4 Ω resistance is 1.2 A, what is
3
2 1 the potential difference between B Voltage across 3 Ω resistance is given
(c) A (d) A and C? [CBSE AIPMT 1994] by Ohm’s law
9 8
4Ω i.e. V = IR = 3 × i 1
Ans. (a) 2Ω = 3 × 0.8 = 2.4 V (as i 1 = 0.8 A)
i1 C
From the given figure, the resistances A 8Ω B i As 3 Ω and 6 Ω are in parallel, hence
in the arms AC and BC are in series.
i2 voltage across 6 Ω resistance will also
∴ R ′ = R AC + R BC
be 2.4 V.
=3 Ω + 3 Ω = 6 Ω
∴ Current through6 Ω resistance
Now, R ′ is in parallel with the resistance V 2.4
i2 = = = 0.4 A
in arm AB, (a) 3.6 V (b) 6.3 V R 6
R ′ × R AB (c) 1.8 V (d) 2.4 V ∴ Total current in the circuit,
So, R=
R ′ + R AB Ans. (a) i = i 1 + i 2 = 0.8 + 0.4 = 1.2 A
6Ω ×3Ω ∴ Potential drop across 4 Ω resistance,
= =2Ω i1= 1.2 A
6Ω +3Ω = i × 4 = 1.2 × 4 = 4.8 V
4Ω 2Ω i
A C
Therefore, the Ohm’s law can be stated 8Ω B
as
87 Kirchhoff ’s first law of electricity
V i2 follows [CBSE AIPMT 1992]
V = i R or current, i =
R (a) only law of conservation of energy
Substituting the values, we get (b) only law of conservation of charge
2 (c) law of conservation of both
i = = 1A (QV = 2 V )
2 The potential difference across4 Ω energy and charge
resistance is given by Ohm’s law (d) sometimes law of conservation of
84 Kirchhoff’s first law, i.e. Σi = 0 at a = 4 × i1 energy and some other times law
= 4 × 1.2 =4.8 V (as i 1 = 1.2 A) of conservation of charge
junction, deals with the
conservation of [CBSE AIPMT 1997] As resistances 4 Ω and 8 Ω are in Ans. (b)
parallel, so potential difference across Kirchhoff’s first law supports law of
(a) angular momentum
8 Ω resistance will also be 4.8 V. conservation of charge. This is because
(b) linear momentum
∴ Current through 8 Ω resistance a point in a circuit cannot act as a
(c) energy V 4.8 source or sink of charge.
(d) charge i2 = = = 0.6 A
R 8
Ans. (d) ∴ i = i 1 + i 2 = 1.2 + 0.6 = 1.8 A 88 A battery of emf 10 V and internal
Kirchhoff ’s law states that the algebraic ∴ Potential difference across2 Ω resistance 0.5 Ω is connected
sum of currents meeting at a point (say resistance across a variable resistance R. The
at a junction) is equals to zero i.e.,
VBC = i × 2 value of R for which the power
Σi = 0
= 1.8 × 2 = 3.6 V
delivered in it is maximum, is Ans. (a) . × 10−1 m
(a) 10 . × 10−1 m
(b) 15
given by [CBSE AIPMT 1992] Applying Kirchhoff’s voltage law in the . × 10−2 m
(c) 15 . × 10−2 m
(d) 10
(a) 0.5 Ω (b) 1.0 Ω given loop and going in direction of Ans. (a)
current PSTQ total voltage is equal to
(c) 2.0 Ω (d) 0.25 Ω According to the question, the metre
zero
Ans. (a) bridge is shown below,
1Ω 4V i 8V 2Ω R 10 Ω
According to maximum power transfer P Q
theorem, the power output across load r1 r2
due to a cell or battery is maximum, if
the load resistance is equal to the 9Ω G
internal resistance of cell or battery. S T x1 x2
Hence, value of R will be0.5 Ω. R
100 cm
89 Two identical batteries each of So, −2i + 8 − 4 − 1 × i − 9i = 0 x1 3
1 Given, =
emf 2 V and internal resistance 1 Ω ∴ i= A x2 2
3
are available to produce heat in an 1 At balance condition in metre bridge,
Potential difference across PQ = ×9
external resistance by passing a 3 R x1
=
current through it. The maximum =3V 10 x 2
power that can be developed TOPIC 4 x1 3
⇒ R= × 10 = × 10 = 15 Ω
across R using these batteries is x2 2
[CBSE AIPMT 1990] Measuring Instruments Now, length of given wire whose
(a) 3.2 W (b) 2 W resistance 15 Ω is 1.5 m.
8
(c) 1.28 W (d) W 91 In a potentiometer circuit, a cell of Therefore, length of 1Ω resistance wire is
9 emf 1.5 V gives balance point at 36 1. 5 1
= = = 0.1 = 1 × 10 −1 m
Ans. (b) cm length of wire. If another cell 15 10
To receive maximum current, the two of emf 2.5 V replaces the first cell, Hence, correct option is (a).
batteries should be connected in series. then at what length of the wire,
Given, R = 1 Ω + 1 Ω = 2 Ω. the balance point occurs? 93 The meter bridge shown in the
P l
Hence, power developed across the [NEET 2021] balance position with = 1 . If we
resistance R with the batteries in series (a) 60 cm (b) 21.6 cm Q l2
is (c) 64 cm (d) 62 cm now interchange the positions of
 E 
2
 2E  Ans. (a) galvanometer and cell, will the
P = i2R =   R I = 
 R + 2r   R eq 
 Given, length of balancing point,L1 = 36 bridge work? If yes, that will be
 2 × 2 cm balanced condition?
=  × 2= 2 W [Qr = 1Ω ]
 2 + 2 E 1 =1.5V [NEET (Odisha) 2019]
E 2 = 2.5V
Note
We know that,
In case of batteries connected in series,
E
equivalent emf is given by E eq = E 1 + E 2 Potential gradient = P Q
L
90 Two batteries of emf 4 V and 8 V E1 E2
Q = G
L1 L2
with internal resistance 1 Ω and 2 Ω l2
where,L2 is the length of the balancing l1
are connected in a circuit with a
resistance of 9 Ω as shown in
point for cell of emf E 2 . P l2 − l1
Substituting the values in the above (a) yes, = (b) no, no null point
figure. The current and potential Q l2 + l1
equation, we get
difference between the points P P l P l
1.5 2.5
= ⇒ L2 = 60 cm (c) yes, = 2 (d) yes, = 1
and Q are [CBSE AIPMT 1988] 36 L2 Q l1 Q l2
1Ω 4V 8V 2Ω Ans. (d)
P Q 92 A resistance wire connected in the
r1 r2 For balanced position in a meterbridge
left gap of a metre bridge
balances a 10 Ω resistance in the
9Ω
right gap at a point which divides
R the bridge wire in the ratio 3 : 2. If P Q
1 1 the length of the resistance wire is
(a) A and 3 V (b) A and 4 V G
3 6 1.5 m, then the length of 1 Ω of the
1 1
(c) A and 9 V (d) A and 12 V resistance wire is l1 l2
9 12 [NEET (Sep.) 2020]
P l1 According to question, emf of the cell is resistance r1 as shown in figure. Current
=
Q l2 directly proportional to the balancing E
in wire AB = 0
length i.e. r1 + r
Now, if position of G and cell is
interchanged, E ∝l …(i) r1
Now, in the first case, cells are connected
G
in series to support one another i.e. E0
Net emf = E 1 + E 2 l P r
P Q From Eq. (i), E 1 + E 2 = 50 cm(given) …(ii) A
B
L
Again cells are connected in series in
opposite direction i.e.
E
Net emf = E 1 − E 2
l1 l2 Potential gradient,
From Eq. (i), E 1 − E 2 = 10 …(iii)
Ir  E  r
From Eqs. (ii) and (iii) x= = 0 
The balance condition still remains the L  r1 + r  L
same if the jockey points as the same E 1 + E 2 50 E 5+ 1 6 3
= ⇒ 1 = = =
point as given in the initial condition, for E 1 − E 2 10 E2 5 − 1 4 2 emf produced across E will be given by
which there is no deflection in the  E r l
galvanometer or no current will be
E = x ⋅l =  0 
96 A potentiometer wire has length 4  r1 + r  L
drawn from the cell. Thus, the bridge
will work as usual and balance condition m and resistance8Ω. The
is same, P / Q = l 1 /l 2 resistance that must be connected 98 The resistances in the two arms of
in series with the wire and an the meter bridge are 5 Ω and R Ω,
94 A potentiometer is an accurate and accumulator of emf 2V, so as to get respectively. When the resistance
versatile device to make electrical a potential gradient 1 mV per cm R is shunted with an equal
measurement of EMF because the on the wire is [CBSE AIPMT 2015] resistance, the new balance point
method involves [NEET 2017] (a) 32 Ω (b) 40 Ω (c) 44 Ω (d) 48 Ω is at 1.6 l 1 . The resistance R, is
(a) cells [CBSE AIPMT 2014]
Ans. (a)
(b) potential gradients ()
Given,l = 4 m,
(c) a condition of no current flow 5Ω
R = potentiometer wire resistance = 8 Ω RΩ
through the galvanometer
dV
(d) a combination of cells, Potential gradient = = 1mV/cm
galvanometer and resistances dr
So, for 400 cm, ∆V = 400 × 1 × 10 −3 = 0.4 V G
Ans. (c)
When a cell is balanced against Let a resistor R S connected in series, so
potential drop across a certain length of as A l1 100 –l 1 B
potentiometer wire, no current flows V 2
∆V = × R ⇒ 0.4 = ×8
through the cell R + RS 8+ R (a) 10 Ω (b) 15 Ω (c) 20 Ω (d) 25 Ω
∴emf of cell = potential drop across 16 Ans. (b)
⇒ 8+ R = = 40 ⇒ R = 32 Ω
balance length of potentiometer wire. 0.4 For first case, balanced condition of
So, potentiometer is a more accurate meter bridge will be
97 A potentiometer wire of length L 5 R
device for measuring emf of a cell or no = ...(i)
current flows through the cell during and a resistance r are connected l1 (100 − l1 )
measurement of emf. in series with a battery of e.m.f. Now, by shunting resistance R by an equal
E 0 and a resistance r1 . An resistanceR, new resistance in that arm
R
95 A potentiometer wire is 100 cm unknown e.m.f. is balanced at a become .
2
long and a constant potential length l of the potentiometer wire.
So, new balanced condition will be
difference is maintained across it. The e.m.f. E will be given by 5 R /2
= ...(ii)
Two cells are connected in series [CBSE AIPMT 2015]
1.6 l1 (100 − 1.6 l1 )
first to support one another and LE r E0 r l
(a) 0 (b) . From Eqs. (i) and (ii),
then in opposite direction. The l r1 (r + r1 ) L
1.6 (100 − 1.6 l1 )
balance points are obtained at 50 E0 l LE0 r = ×2
(c) (d) 1 100 − l1
cm and 10 cm from the positive L (r + r1 ) l
⇒ 160 − 1.6 l1 = 200 − 3.2 l1
end of the wire in the two cases. Ans. (b) 1.6 l1 = 40
The ratio of emf is [NEET 2016] 40
Consider a potentiometer wire of length l1 = = 25 m
(a) 5 : 4 (b) 3 : 4 (c) 3 : 2 (d) 5 : 1 L and a resistance r are connected in 1.6
5 R
Ans. (c) series with a battery of emf E 0 and a ⇒From Eq. (i), = ⇒ R = 15 Ω
25 75
99 A potentiometer circuit has been Ans. (b) (a) k (l 2 − l 1 ) and kl 2 (b) kl 1 and k (l 2 − l 1 )
set up for finding the internal Effective resistance, (c) k (l 2 − l 1 ) and kl 1 (d) kl 1 and kl 2
resistance of a given cell. The main Q Ans. (c)
battery, used across the The balancing length for R (when 1, 2 are
10 Ω 30 Ω
potentiometer wire, has an emf of connected) bel 1 and balancing length
2.0 V and a negligible internal P
for R + X (when 1, 3 is connected) isl 2 .
G 50 Ω
R
resistance. The potentiometer wire Then, iR = kl 1 and i (R + X) = kl 2
itself is 4 m long. When the Given, i = 1A
90 Ω 30 Ω ∴™ R = kl 1 …(i)
resistance R, connected across the
given cell, has values of (i) infinity, R + X = kl 2 …(ii)
S Also, subtracting Eq. (i) from Eq. (ii), we
(ii) 9.5 Ω, the balancing lengths,
get
on the potentiometer wire are X = k (l 2 − l 1 )
found to be 3 m and 2.85 m, 7 V, 5 Ω
respectively. 1 1 1 102 4Ω
= + …(i)
The value of internal resistance R eff R 1 R 2
of the cell is [CBSE AIPMT 2014] then, R 1 = 10 + 30
(a) 0.25 Ω (b) 0.95 Ω R 1 = 40 3Ω
P M
(c) 0.5 Ω (d) 0.75 Ω Now, R 2 = 90 + 30 = 120
Ans. (c) R 2 = 120
By Eq. (i), N
0.5Ω 1Ω
Given, e = 2V and l = 4 m 1
=
1
+
1
Potential drop per unit length R eff 40 120 0.5Ω
e 2 40 × 120 4800
φ = = = 0.5 V / m R eff = = = 30 Ω In the circuit shown, the current
l 4 120 + 40 160
Case I
through the 4 Ω resistor is 1 A
In the balancing condition,
e ′ = φl1 ...(i) 7 7 when the points P and M are
∴ Current, I = = = 0.2 A connected to a DC voltage source.
(e ′ = emf of the given cell) (30 + 5) 35
Case II  
The potential difference between
E
V = φl2 ...(ii) QI = R + r 
the pointsM and N is
 [CBSE AIPMT 2008]
From Eqs. (i) and (ii),
(a) 1.5 V (b) 1.0 V (c) 0.5 V (d) 3.2 V
e ′/V = l1 / l2 101 A potentiometer circuit is set up
Ans. (d)
∴ e ′ = I (r + R) as shown. The potential gradient
across the potentiometer wire, is Potential across PM,
and V = IR for the second case
k volt/cm and the ammeter, VPM = 4 × 1 = 4 V
I (r + R) l 1
= present in the circuit, reads 1.0 A Now, equivalent resistance across PN
IR l2 1 1 1
when two way key is switched off. = +
l   3  Req 0.5 0.5
So, r = R  1 − 1 = 9.5  − 1 The balance points, when the key
 l2   2.85  0.5
R eq = = 0.25 Ω
between the terminals (a) 1 and 2
2
= 9.5 (1.05 − 1) (b) 1 and 3, is plugged in, are found
Effective resistance between P and M
= 9.5 × 0.05 = 0.475 −
~ 0.5 Ω to be at lengths l 1 cm and l 2 cm
R PM = 0.25 + 1 = 1.25 Ω
respectively. The magnitudes, of
For balancing condition,
100 The resistances of the four arms the resistors R and X in ohm, are VMN R MN
then equal, respectively to =
P, Q,R and S in a Wheatstone bridge VPM R PM
[CBSE AIPMT 2010]
are 10 Ω, 30 Ω, 30 Ω and 90 Ω, V × R MN
VMN = PN =
1
× 4 = 3.2 V
+ –
respectively. The emf and internal R PM 1.25
resistance of the cell are 7 V and
5 Ω respectively. If the 103. A cell can be balanced against 110
galvanometer resistance is 50 Ω, A B cm and 100 cm of potentiometer
1 G
the current drawn from the cell will 2
wire, respectively with and without
be [NEET 2013] 3 being short circuited through a
(a) 1.0 A resistance of 10 Ω. Its internal
– R X
(b) 0.2 A resistance is [CBSE AIPMT 2008]
(c) 0.1 A + A (a) 1.0 Ω (b) 0.5 Ω
(d) 2.0 A (c) 2.0 Ω (d) zero
Ans. (a) 105 Five equal resistances each of equivalent resistance of the
In potentiometer experiment in which resistance R are connected as combination as seen by the
we find internal resistance of a cell. Let shown in the figure. A battery of battery is [CBSE AIPMT 2003]
E be the emf of the cell andV be the
terminal potential difference, then
4V volts is connected between A (a) R (b) 2 R
and B. The current flowing in R R
E l1 (c) (d)
= AFCEB will be [CBSE AIPMT 2004] 4 2
V l2
C Ans. (a)
where,l 1 and l 2 are lengths of R AB R AD
potentiometer wire with and without being =
R R BC R DC
short circuited through a resistance. R R
E R+r As bridge is in balanced condition, no
Since, = F
V R R A current will flow through BD.
B
[QE = I (R + r) and V = IR] B
R + r l1 r 110 D E
∴ = or 1 + = R R R
R l2 R 100 3V V V 2V
(a) (b) (c) (d)
r 10 1 R R 2R R A R G C
or = or r = × 10 = 1 Ω
R 100 10
Ans. (c)
R R
104 Three resistance P, Q, R each of 2 Ω The given circuit can be redrawn as
shown. D
and an unknown resistance S form FC FD
the four arms of a Wheatstone From circuit, = =1
CE DE
bridge circuit. When a resistance R 1 = R AB + R BC
C
of 6Ω is connected in parallel to S =R+ R
the bridge gets balanced. What is =2R
R R
the value of S ? [CBSE AIPMT 2007] R 2 = R AD + R CD
(a) 2 Ω (b) 3 Ω = R + R =2R
A R B
(c) 6 Ω (d) 1 Ω F E R 1 and R 2 are in parallel combination.
Ans. (b) R R Hence, equivalent resistance between
A and C will be
Concept Balancing condition of
D R1R2 4R 2
Wheatstone bridge is used to calculate ∴ R eq = = =R
the value of unknown resistance. R 1 + R 2 4R
Thus, it is balanced Wheatstone bridge,
The situation can be depicted as shown so, resistance in armCD is ineffective
in figure. and so no current flows in this arm. 107 Resistivity of potentiometer wire
A Net resistance of the circuit is is 10 −7 Ω-m and its area of
1
=
1
+
1 cross-section is 10 −6 m 2 . When a
P Q R ′ (R + R) (R + R) current i = 0.1 A flows through the
2Ω 2Ω 1 1 2 1
= + = = wire, its potential gradient is
2R 2R 2R R [CBSE AIPMT 2001]

2Ω ∴ R′ = R (a) 10−2 V/m (b) 10−4 V/m


S
So, net current drawn from the battery (c) 0.1 V/m (d) 10 V/m
R
is Ans. (a)
6Ω
V V
B i′ = = Potential gradient
R′ R V
= Potential fall per unit length =
As from symmetry, upper circuit AFCEB l
is half of the whole circuit and is equal = current × resistance per unit length
As resistances S and 6 Ω are in parallel to AFDEB. R
=i ×
their effective resistance is So, in both the halves half of the total l
6S current will flow. ρl R ρ
Ω but R= ⇒ =
6+ S Hence, in AFCEB, the current flowing is A l A
i′ V where, symbols have their usual
As the bridge is balanced, hence it is i= =
2 2R meanings.
balanced Wheatstone bridge. ρ
For balancing condition, ∴ Potential gradient = i ×
2 2 (6 + S ) A
P
=
R
or = 106 In a Wheatstone bridge, all the
Here,ρ = 10 −7 Ω -m , i = 0.1 A
Q  6S  2 6S four arms have equal resistance R.
  and A = 10 −6 m 2
6 + S  If the resistance of the
or 3 S = 6 + S or S = 3 Ω galvanometer arm is also R, the
10 −7 (a) it has a wire of high resistance R and S are also in series, so their
Hence, potential gradient = 0.1 ×
10 −6 (b) it has a wire of low resistance equivalent resistance = 6 + 8 = 14 Ω
=
0.1
= 0.01 = 10 −2 V / m (c) it does not draw current from Now, 7 Ω and 14 Ω resistances are in
10 external circuit parallel, so
(d) it draws a heavy current from 7 × 14 7 × 14 14
R AB = = = Ω
108 In a Wheatstone bridge resistance external circuit 7 + 14 21 3
of each of the four sides is 10 Ω. If Ans. (c) Note Normally, in Wheatstone bridge
the resistance of the When we measure the emf of a cell by in middle arm galvanometer must be
galvanometer is also 10 Ω, then the potentiometer then no current is connected. In Wheatstone bridge, cell
effective resistance of the bridge drawn from the external circuit. Thus, in and galvanometer arms are
this condition the actual value of a cell interchangeable.
will be [CBSE AIPMT 2001]
In both the cases, condition of
is found. In this way potentiometer is
(a) 10 Ω (b) 5 Ω balanced bridge is
equivalent to an ideal voltmeter of
(c) 20 Ω (d) 40 Ω P R
infinite resistance. =
Ans. (a) Q S
The given circuit can be shown as, 110 A bridge circuit is shown in figure.
111 A potentiometer consists of a wire
The equivalent resistance
B of length 4 m and resistance 10 Ω.
between A and B will be It is connected to a cell of emf 2 V.
10 Ω [CBSE AIPMT 2000]
10 Ω The potential gradient of the wire is
P Q F [CBSE AIPMT 1999]
A C
10 Ω
3Ω 4Ω (a) 0.5 V/m b) 2 V/m
R S (c) 5 V/m (d) 10 V/m
10 Ω 10 Ω E G
A 7Ω B Ans. (a)
Potential applied
D
6Ω 8Ω Potential gradient =
Length of wire
P 10 V
From figure, = =1 H ∴ k=
Q 10 l
252 14
R 10
= =1 (a) 21 Ω (b) 7 Ω (c) Ω (d) Ω Given, V = 2 V, l = 4 m
85 3 2
S 10 ∴ k = = 0.5 V/m
P R 4
∴ = Ans. (d)
Q S The bridge circuit can be shown as 112 In meter bridge, the balancing
Hence, Wheatstone bridge is balanced. F length from left is found to be 20
Therefore, the galvanometer will be cm when standard resistance of
ineffective. 3Ω 4Ω
P Q 1 Ω is in right gap. The value of
The above Wheatstone bridge can be
A
E
7Ω
G unknown resistance is
redrawn as B
[CBSE AIPMT 1999]
10 Ω 10 Ω R S (a) 0.25 Ω (b) 0.4 Ω (c) 0.5 Ω (d) 4 Ω
6Ω 8Ω
P Q Ans. (a)
A C H Let unknown resistance be X.
R S The balanced condition of bridge circuit is X R=1Ω
given by
10 Ω 10 Ω P 3 R 6 3 P R
= , = = ⇒ =
Resistances P and Q are in series, so Q 4 S 8 4 Q S
R ′ = 10 + 10 = 20 Ω Thus, it is balanced Wheatstone bridge,
Resistances R and S are in series, so so potential at F is equal to potential at
H. Therefore, no current will flow P = 20 Ω Q = 80 Ω
R ′ ′ = 10 + 10 = 20 Ω
through 7Ω resistance. So, circuit can 20 cm (100 – 20) cm
Now, R ′ and R ′ ′ are in parallel hence, be redrawn as
net resistance of the circuit P=3Ω Q=4Ω Then condition of Wheatstone bridge
X 20
R ′ × R ′ ′ 20 × 20 gives, =
= = = 10 Ω P Q R 80
R ′ + R ′ ′ 20 + 20
A B where, X = unknown resistance
R S R = known resistance
109 Potentiometer measures the Hence, unknown resistance is
potential difference more R=6Ω S=8Ω 20 1
X= ×R= ×1 (QR = 1 Ω )
accurately than a voltmeter, P and Q are in series, so their equivalent 80 4
because [CBSE AIPMT 2000] resistance = 3 + 4 = 7 Ω = 0.25 Ω

You might also like